حمید آنالیز

Well-Known Member
ارسال ها
1,351
لایک ها
1,322
امتیاز
113
#81
پاسخ : ماراتن هندسه

ببخشید من اثبات شما رو متوجه نشدم! الان نقاط روی اضلاع که مرکز دایره ی محاطی مثلثشون بر مرکز دایره ی محاطی مثلث
منطبقه
هستند؟ اگه اینجوریه چرا
هست؟ این اتفاق فقط برای نقطه هایی که دایره ی محاطی بر اضلاع
در اون نقاط مماسه رخ میده!

---- دو نوشته به هم متصل شده است ----

من سوال بعدو میذارم تا بتونیم ماراتن رو سریع تر پیش ببریم:
یک متوازی الاضلاع است. دایره ی با قطر
،
را در
قطع می کند. عمودی که از
بر
رسم می شود
را به ترتیب در
قطع می کند. ثابت کنید نقاط
روی یک دایره قرار دارند.
یه سوال اون دایرو مماس قطر هاست؟؟؟!!!!:13:
درون متوازی الاضلاع است یا نه؟!!!:4:
 
ارسال ها
288
لایک ها
154
امتیاز
43
#82
پاسخ : ماراتن هندسه

یعنی دایره ای که مرکزش وسط قطر ad است .

---- دو نوشته به هم متصل شده است ----

یعنی دایره ای که مرکزش وسط قطر AD است .
 

J.Karimi

Active Member
ارسال ها
169
لایک ها
115
امتیاز
43
#83
پاسخ : ماراتن هندسه

با اين كه سوال قبل جواب داده نشده ولي اين سوال رو گفتم اينجا مطرح كنم بهتر باشه تا يه تايپيك ديگه!!!

درمثلث حاده الزاويه
اگر
نيم ساز باشد و O مركز دايره محيطي فرض كنيد محل برخورد ارتفاع نظير راس C و خط


روي دايره محيطي
واقع شود زاويه ي
رابيابيد
 

Dadgarnia

New Member
ارسال ها
1,350
لایک ها
1,127
امتیاز
0
#84
پاسخ : ماراتن هندسه

با اين كه سوال قبل جواب داده نشده ولي اين سوال رو گفتم اينجا مطرح كنم بهتر باشه تا يه تايپيك ديگه!!!

درمثلث حاده الزاويه
اگر
نيم ساز باشد و O مركز دايره محيطي فرض كنيد محل برخورد ارتفاع نظير راس C و خط


روي دايره محيطي
واقع شود زاويه ي
رابيابيد
محل برخورد این دو خط رو
می نامیم. داریم:

حالا ثابت می کنیم برای هر چهار عدد حقیقی و مثبت مثل
که
داریم
. داریم:


پس این چهار عدد ریشه ی معادله
هستند پس
. حالا اگه توی مثلث های
قضیه کسینوس ها رو بنویسیم بدست میاد:


 

Sharifi_M

New Member
ارسال ها
561
لایک ها
348
امتیاز
0
#85
پاسخ : ماراتن هندسه

دو دایره با شعاع برابر r، یکی از رئوس A و B و دیگری از رئوس B و C متوازی الاضلاع ABCD میگذرند. M نقطه برخورد دوم این دو دایره است. ثابت کنید شعاع دایره محیطی مثلث AMD برابر است با r.
 

Dadgarnia

New Member
ارسال ها
1,350
لایک ها
1,127
امتیاز
0
#86
پاسخ : ماراتن هندسه

دو دایره با شعاع برابر r، یکی از رئوس A و B و دیگری از رئوس B و C متوازی الاضلاع ABCD میگذرند. M نقطه برخورد دوم این دو دایره است. ثابت کنید شعاع دایره محیطی مثلث AMD برابر است با r.
مراکز دایره محیطی مثلث های
رو به ترتیب
و قرینه ی
نسبت به
رو
می نامیم. واضحه که
. داریم:




پس
مرکز دایره محیطی مثلث
هست و شعاعش برابره با
پس حکم ثابت شد.

---- دو نوشته به هم متصل شده است ----

در مثلث
داریم
و
. نقاط
به ترتیب روی
طوری قرار دارند که
ثابت کنید
.
 

assassin1

New Member
ارسال ها
80
لایک ها
18
امتیاز
0
#87
پاسخ : ماراتن هندسه

سلام صورت سوالو مطمینید مشکلی نداره؟؟؟!!
 

Sharifi_M

New Member
ارسال ها
561
لایک ها
348
امتیاز
0
#88
پاسخ : ماراتن هندسه

در مثلث
داریم
و
. نقاط
به ترتیب روی
طوری قرار دارند که
ثابت کنید
.
من نمیدونم یچنین روش حل یک مساله هندسه درسته یا نه. :/

اگر
حکم برقرار میشه.
بعد میشه از حکم برسیم به فرض؟ یعنی فرض کنیم تساوی آخر برقراره و مثلث متساوی الساقینه با بقیه شرط ها بجر اینکه زاویه راس 20 درجست، و از این برسیم به اینکه راس زاویش 20 عه.
اگه این کار درسته دیگه بقیش حله!
 

Dadgarnia

New Member
ارسال ها
1,350
لایک ها
1,127
امتیاز
0
#89
پاسخ : ماراتن هندسه

سلام صورت سوالو مطمینید مشکلی نداره؟؟؟!!
بله درسته.

---- دو نوشته به هم متصل شده است ----

من نمیدونم یچنین روش حل یک مساله هندسه درسته یا نه. :/

اگر
حکم برقرار میشه.
بعد میشه از حکم برسیم به فرض؟ یعنی فرض کنیم تساوی آخر برقراره و مثلث متساوی الساقینه با بقیه شرط ها بجر اینکه زاویه راس 20 درجست، و از این برسیم به اینکه راس زاویش 20 عه.
اگه این کار درسته دیگه بقیش حله!
فرض
رو نمي توني اضافه كني مگه اينكه يكي از فرض هاي مربوط به
رو حذف كني اونجوري فكر نكنم سوال خيلي فرق كنه.
 

assassin1

New Member
ارسال ها
80
لایک ها
18
امتیاز
0
#92
پاسخ : ماراتن هندسه

شما شكلتونو اشتباه كشيدين. زاويه بيست درجه خيلي كوچيكه. با جئوجبرا امتحان كنيد درست در مياد.
راستش نتونستم راه حلمو بنویسم ولی تو این جور سوالات باید یه سری شکل بسازیم دراین سوال:روی ضلع bcمثلث متساوی الاضلاع bcfرا می سازیم و نقطه fرا به
m,nوصل می کنیم سپس با همیهشتی دو مثلث fcM وaNd و استفاده از اینکه adfb ذوزنقه ی متساوی الساقین است زاویه dبه صورت دو زاویه 20 درجه بدست میاد پس میشه 40 درجه .آیا حل شما هم اینطوره؟!
 
آخرین ویرایش توسط مدیر

assassin1

New Member
ارسال ها
80
لایک ها
18
امتیاز
0
#93
پاسخ : ماراتن هندسه

شما شكلتونو اشتباه كشيدين. زاويه بيست درجه خيلي كوچيكه. با جئوجبرا امتحان كنيد درست در مياد.
اگه توضیح واضح نیست بگید تا شکلو بکشم:confused:

---- دو نوشته به هم متصل شده است ----

سوال:در مثلث متساوی الساقین
اندازه زاویه
برابر 80 درجه است.نقطه
را درون این مثلث چنان انتخاب می کنیم که
. مقدار زاویه
را بیابید.
 

Dadgarnia

New Member
ارسال ها
1,350
لایک ها
1,127
امتیاز
0
#94
پاسخ : ماراتن هندسه

راستش نتونستم راه حلمو بنویسم ولی تو این جور سوالات باید یه سری شکل بسازیم دراین سوال:روی ضلع bcمثلث متساوی الاضلاع bcfرا می سازیم و نقطه fرا به
m,nوصل می کنیم سپس با همیهشتی دو مثلث fcd وaed و استفاده از اینکه adfb ذوزنقه ی متساوی الساقین است زاویه dبه صورت دو زاویه 20 درجه بدست میاد پس میشه 40 درجه .آیا حل شما هم اینطوره؟!
چيٓن؟ نه حل من اينجوري نبود.

---- دو نوشته به هم متصل شده است ----

نمي خواين سوالتونو عوض كنيد؟ يه زاويه بازي ساده ست.
 

assassin1

New Member
ارسال ها
80
لایک ها
18
امتیاز
0
#95
پاسخ : ماراتن هندسه

ببخشید اصلاح شد. خب راستش سوال ندارم چیکار کنم
 

Dadgarnia

New Member
ارسال ها
1,350
لایک ها
1,127
امتیاز
0
#96
پاسخ : ماراتن هندسه

ببخشید اصلاح شد. خب راستش سوال ندارم چیکار کنم
هنوز هم
توش هست :| ولي اشكالي نداره من ايده ي شما رو چك كردم و ديدم كه به جواب ميرسه. پس من سوال بعد رو ميذارم:
دو دايره
در نقاط
متقاطع اند. خط دلخواهي از
مي گذرد و
را به ترتيب در
(به غير از
) قطع مي كند. پاي عمود هاي وارد از
بر مماس هايي كه از نقاط
به ترتيب بر
رسم مي شوند را
مي ناميم. ثابت كنيد
بر دايره به قطر
مماس است.
 
آخرین ویرایش توسط مدیر

Sharifi_M

New Member
ارسال ها
561
لایک ها
348
امتیاز
0
#97
پاسخ : ماراتن هندسه

اینم یخورده خیلی آسون نیست؟!
M روی نیمسازه مثلثه و بقیش زاویه بازی و همنهشتی


بعدا نوشت: فکر کردم طراح سوال سوالشو اصلاح کرده :-"
 
آخرین ویرایش توسط مدیر

Sharifi_M

New Member
ارسال ها
561
لایک ها
348
امتیاز
0
#99
پاسخ : ماراتن هندسه

همزمان پست دادیم تقریبا! :4:
 

REZA 73

Active Member
ارسال ها
139
لایک ها
184
امتیاز
43
پاسخ : ماراتن هندسه

هنوز هم
توش هست :| ولي اشكالي نداره من ايده ي شما رو چك كردم و ديدم كه به جواب ميرسه. پس من سوال بعد رو ميذارم:
دو دايره
در نقاط
متقاطع اند. خط دلخواهي از
مي گذرد و
را به ترتيب در
(به غير از
) قطع مي كند. پاي عمود هاي وارد از
بر مماس هايي كه از نقاط
به ترتيب بر
رسم مي شوند را
مي ناميم. ثابت كنيد
بر دايره به قطر
مماس است.
نقطه تقاطع خط دلخواهی که از
میگذشت با دایره به قطر
را
فرض کنید. خط
در نقطه
بر دایره مفروض مماس است.
برای ثابت کردن مماسی هم کافی است ثابت کنید چهارضلعی
و چهارضلعی
محاطی هستند.
 
آخرین ویرایش توسط مدیر
بالا